Part (a) of Theorem 2.27 in Rudin's POMA

  • Thread starter math771
  • Start date
  • Tags
    Theorem
In summary, Rudin's proof claims that if p does not belong to the closure of E, then it cannot have a neighborhood that intersects the closure of E. However, it is shown that if p does not belong to the closure of E, then its same neighborhood also does not intersect the closure of E.
  • #1
math771
204
0
The theorem reads: If X is a metric space and E is a subset of X, then the closure of E is closed.
Rudin's proof: If p belongs to X and not to the closure of E, then p is neither a point of E nor a limit point of E. Hence p has a neighborhood which does not intersect E. The complement of the closure of E is therefore open. Hence the closure of E is closed (POMA, 35).
I'm having trouble seeing why the italicized portion of the proof is correct. Yes, p has a neighborhood which does not intersect E, but, (if I'm not mistaken) the statement that the complement of the closure of E is open does not follow from the fact that p has such a neighborhood. p, it seems, would need to have a neighborhood which does not intersect the closure of E (and not just E itself), for a neighborhood of p that contains limit points of E but not any points of E itself would not be a subset of the complement of the closure of E (in which case, p would not be an interior point of the complement of the closure of E).
Now, intuitively, it seems that if p has a neighborhood which does not intersect E, it cannot have one that intersects the set of limit points of E. However, the fact that Rudin does not mention this statement suggests that there is something about his proof that I do not understand.
Thanks in advance for any help!
 
Physics news on Phys.org
  • #2
Hi math771! :smile:

You are indeed correct, just because p has a neighborhood that doesn't intersect E doesn't mean that the complement of the closure is open.

However, I claim that if p has a neighborhood that doesn't intersect E, then that same neigborhood also doesn't intersect the closure of E. Can you show this?
 
  • #3
I think so. I'll try to prove the contrapositive statement that if a neighborhood of p, called N, intersects the closure of E, then it must intersect E. This is equivalent to showing that if N intersects the set of limit points of E, it must intersect E. If N (of radius r) contains a limit point of E, called q, then d(p,q)<r. Therefore, r - d(p,q)>0. So, we can consider a neighborhood of q of radius r - d(p,q)>0. Because q is a limit point of E, there is a point q' belonging to E such that d(q,q')<r - d(p,q). Thus, d(p,q')<(or=) d(p,q) + d(q,q')<r. So, q' belongs to N.
Hopefully, that's all clear. Would appreciate corrections/suggestions. Thanks!
 
  • #4
Sounds good! :smile:

One very small remark though. You first with an arbitrary neighborhood N around p, and then later in the proof you assumed that this neighborhood is actually a ball of radius r. It might be worth thinking about why you can take the neighborhood to be a ball. You probably know already, but I was just pointing it out for if you didn't :smile:
 
  • #5
Because all the points we're talking about are contained in a metric space?
 
  • #6
Yes, but why can we use balls instead of neighborhoods in a metric space? Not every neighborhood is a ball!
 
  • #7
Because the definition of distance between two points in a euclidean space implies that neighborhoods in that space are balls?
 
  • #8
math771 said:
Because the definition of distance between two points in a euclidean space implies that neighborhoods in that space are balls?

No, this is certainly not true. There are a lot of neighboorhoods that aren't balls in general. Take [itex]\mathbb{R}[/itex], then

[tex]]-1,1]\cup \{81\}[/tex]

is a neighborhood of 0. But this is certainly not a ball!
 
  • #9
Rudin simply defines a "neighborhood of a point p" as "a set N-sub-r(p) consisting of all points q such that d(p,q)<r," where r is the radius of the neighborhood (32). According to this definition, the set that you present would not be a neighborhood of 0. In fact, it would not be a neighborhood at all, for you can't choose any point p such that the set contains all q obeying the equation d(p,q)<r for some r.
 
  • #10
Hmm, I checked Rudin and it's true. What a weird thing to do, it's the first time I saw anybody do that...

Oh well, that makes your proof 100% correct! Sorry for the confusion! :blushing:
 
  • #11
Okay, thank you!
(I see that the definition of a neighborhood is quite different on wikipedia :smile:)
 

Related to Part (a) of Theorem 2.27 in Rudin's POMA

1. What is Part (a) of Theorem 2.27 in Rudin's POMA?

Part (a) of Theorem 2.27 in Rudin's POMA states that if a sequence of real numbers is increasing and bounded above, then it converges to its supremum.

2. What does it mean for a sequence to be increasing?

A sequence is considered increasing if each term in the sequence is greater than or equal to the previous term.

3. What does it mean for a sequence to be bounded above?

A sequence is considered bounded above if there exists a number that is greater than or equal to all of the terms in the sequence.

4. What is the supremum of a sequence?

The supremum of a sequence is the smallest number that is greater than or equal to all of the terms in the sequence.

5. Can you provide an example that illustrates Part (a) of Theorem 2.27?

Sure, consider the sequence {1, 2, 3, 4, 5}. This sequence is increasing and bounded above by 5. Therefore, by Part (a) of Theorem 2.27, it converges to its supremum, which is 5.

Similar threads

  • Topology and Analysis
Replies
5
Views
236
Replies
9
Views
3K
Replies
11
Views
3K
  • Topology and Analysis
Replies
1
Views
797
  • Topology and Analysis
Replies
5
Views
1K
  • Topology and Analysis
Replies
4
Views
2K
  • Topology and Analysis
Replies
12
Views
2K
  • Topology and Analysis
Replies
5
Views
2K
  • Topology and Analysis
Replies
7
Views
2K
  • Topology and Analysis
2
Replies
44
Views
5K
Back
Top